Đến nội dung

nthoangcute nội dung

Có 984 mục bởi nthoangcute (Tìm giới hạn từ 28-04-2020)



Sắp theo                Sắp xếp  

#344612 CMR: $(a+\frac{b}{x})^4+(a+\frac{b...

Đã gửi bởi nthoangcute on 08-08-2012 - 08:55 trong Bất đẳng thức và cực trị

Cho $$a,b$$ dương và $$x,y,z$$ dương với $$x+y+z=1$$. Chứng minh rằng:

$$\left(a+\dfrac{b}{x}\right)^4+\left(a+\dfrac{b}{y}\right)^4+\left(a+\dfrac{b}{z}\right)^4\geq 3\left(a+3b\right)^4$$

Áp dụng BĐT:
$$m^4+n^4+p^4 \geq \dfrac{\left(m+n+p\right)^4}{27}$$
Ta được:
$$\left(a+\dfrac{b}{x}\right)^4+\left(a+\dfrac{b}{y}\right)^4+\left(a+\dfrac{b}{z}\right)^4$$
$$\geq \dfrac{\left(3a+\dfrac{b}{x}+\dfrac{b}{y}+\dfrac{b}{z}\right)^4}{27}$$
$$\geq \dfrac{\left(3a+\dfrac{9b}{x+y+z}\right)^4}{27}$$
$$=\dfrac{\left(3a+9b\right)^4}{27}$$
$$=3\left(a+3b\right)^4$$



#345030 Vật thể bay không xác định- NASA

Đã gửi bởi nthoangcute on 09-08-2012 - 10:32 trong Góc giao lưu

Đình chiến đi !
__________________________________________
Ý kiến của mình là:
1. Tin UFO là có thật (tuy tớ hỏi Phạm Tuân về cái này, ông ấy bảo là không có thật)
2. Nhưng mình thấy trong video ấy và cả trong part 2 của video ấy nữa thì một số cái là do lỗi kĩ thuật hoặc UFO thực chất chỉ là một hiệu ứng gây ra bởi các hạt từ tính vũ trụ lên bộ cảm biến hoặc thiết bị tích điện kép của máy ảnh.
3. Một số cái đúng là đáng ngờ thật đó !



#345079 Chứng minh $\frac{a^2b}{2a+b}+\frac{b...

Đã gửi bởi nthoangcute on 09-08-2012 - 13:50 trong Bất đẳng thức và cực trị

Bài 1 Cho a,b,c là các số thực dương và $a+b+c=3$ chứng minh bất đẳng thức sau
$$\frac{a^2b}{2a+b}+\frac{b^2c}{2b+c}+\frac{c^2a}{2c+a} \le 1$$

Chém bài 1 nhanh:
Áp dụng UTC ta có:
$\frac{a^2b}{2a+b} \leq \frac{a^2+2ab}{9} \Leftrightarrow \frac{-2a(a-b)^2}{9(2a+b)} \geq 0$ luôn đúng
Suy ra $\frac{a^2b}{2a+b}+\frac{b^2c}{2b+c}+\frac{c^2a}{2c+a}\leq \frac{(a+b+c)^2}{9}=1$



#345082 Giải phương trình: $3^{3+3x} +3^{3-2x} +3^{4-x...

Đã gửi bởi nthoangcute on 09-08-2012 - 13:56 trong Phương trình - hệ phương trình - bất phương trình

Hình như đề cũ đúng đó bạn... có 2 nghiệm rất đẹp: $1$ và $-\sqrt{2}$
(có điều mình đang giải, đáp số đã đc thử.. ^^ ,)

Sai rồi cậu à !
1 không phải là nghiệm mà 0,9992396116 mới là nghiệm !
$-\sqrt{2}$ không phải là nghiệm mà -1,411788153 mới là nghiệm !



#345108 Tìm Min A=$= \frac{1}{a^{2}+b^{2...

Đã gửi bởi nthoangcute on 09-08-2012 - 15:17 trong Bất đẳng thức và cực trị

1. Cho a,b,c >0 và a+b+c$\leq 1$ , tìm gtnn của :
$A= \frac{1}{a^{2}+b^{2}+c^{2}} +\frac{1}{ab}+\frac{1}{bc}+\frac{1}{ca}$
$B= \frac{1}{a^{2}+bc}+\frac{1}{b^{2}+ca}+\frac{1}{c^{2}+ab}+\frac{1}{ab}+\frac{1}{bc}+\frac{1}{ca}$

a) $A= \frac{1}{a^{2}+b^{2}+c^{2}} +\frac{1}{ab}+\frac{1}{bc}+\frac{1}{ca}$
$=\frac{1}{a^{2}+b^{2}+c^{2}} +\frac{1}{2ab}+\frac{1}{2bc}+\frac{1}{2ca}+\frac{1}{2ab}+\frac{1}{2bc}+\frac{1}{2ca}$
$\geq \frac{9}{(a+b+c)^2}+\frac{1}{2abc}$
$\geq 30$
$A_{min}=30$ khi và chỉ khi $a=b=c=\frac{1}{3}$
b) $B= \frac{1}{a^{2}+bc}+\frac{1}{b^{2}+ca}+\frac{1}{c^{2}+ab}+\frac{1}{ab}+\frac{1}{bc}+\frac{1}{ca}$
$\geq \frac{16}{(a+b+c)^2}=16$
___________
P/s: Post chậm !



#345115 Có thể viết được số $2011$ không?

Đã gửi bởi nthoangcute on 09-08-2012 - 15:35 trong Tổ hợp và rời rạc

Bài toán. Cho $x=1,y=2$. Thực hiện trò chơi như sau. Từ hai số $x$ và $y$ được phép viết $x+y+xy$. Hỏi có thể viết được số $2011$ không?

Em đoán là không được ! Thử làm như này xem đúng không ?
Ta có:
$2011$ phải viết dưới dạng $m+n+mn$
Suy ra $\{m = 0, n = 2011\}, \{m = 1, n = 1005\}, \{m = 3, n = 502\}, \{m = 502, n = 3\}, \{m = 1005, n = 1\}, \{m = 2011, n = 0\}$
Vậy ta phải tìm được số $1005$
Tương tự với số $1005$, ta phải tìm được số $502$
Xét phương trình nghiệm nguyên $502=a +b+ab$
Ta tìm được $\{a = 0, b = 502\}, \{a = 502, b = 0\}$
Suy ra vô lý
Vậy không thể tìm được !



#345131 Tìm các giá trị của $x$ để $P\left ( x \right )=2002...

Đã gửi bởi nthoangcute on 09-08-2012 - 16:23 trong Đại số

Bài toán: Cho đa thức $P\left ( x \right )$ với hệ số nguyên. Biết rằng $P\left ( x \right )$ nhận giá trị bằng $1$ với $6$ giá trị nguyên phân biệt của $x$. Tìm các giá trị của $x$ để $P\left ( x \right )=2002$.

Từ giả thiết ta có:
$P(x)=((x-x_1)(x-x_2)(x-x_3)(x-x_4)(x-x_5)(x-x_6)-1)G(x)$
Vậy $P(x)=2002$ khi và chỉ khi:
$((x-x_1)(x-x_2)(x-x_3)(x-x_4)(x-x_5)(x-x_6)-1)G(x)=2002$
Suy ra $(x-x_1)(x-x_2)(x-x_3)(x-x_4)(x-x_5)(x-x_6)-1 \in \{\pm 1,\pm 2,\pm 7,\pm 11,\pm 13,\pm 14,\pm 22,\pm 26,\pm 77,\pm 91,\pm 143,\pm 154,\pm 182,\pm 286,\pm 1001,\pm 2002 \}$
Hay $a_1a_2a_3a_4a_5a_6-1 \in
\{\pm 1,\pm 2,\pm 7,\pm 11,\pm 13,\pm 14,\pm 22,\pm 26,\pm 77,\pm 91,\pm 143,\pm 154,\pm 182,\pm 286,\pm 1001,\pm 2002 \}$
với $a_1,a_2,a_3,a_4,a_5,a_6$ nguyên và phân biệt
Do đây là phương trình nghiệm nguyên !
Giả sử có số $k$ trong tập hợp trên, khi đó $a_1a_2a_3a_4a_5a_6=k+1$
Mà $a_1,a_2,a_3,a_4,a_5,a_6$ nguyên và phân biệt
Suy ra $k+1$ khi phân tích thành thừa số thì phải có ít nhất 4 nhân tử khác 1 và -1 !
Chỉ có duy nhất một số trong tập hợp trên thỏa mãn là $k=-1001$ thôi (thử là biết)
Suy ra $P(x)=-1001G(x)$
Suy ra $G(x)=-2$ với mọi $x$
Nhưng do tồn tại $x=x_1$ để $P(x)=1$ hay $G(x)=-\frac{1}{1001}$
Từ đó suy ra vô lý
__________________________________________
P/s: Tôi nhớ đề là khác cơ, bởi vì chính bài này đã hạ nhục tôi !



#345135 Chứng minh rằng: $\sum \frac{a^2+b^2}{a+b}...

Đã gửi bởi nthoangcute on 09-08-2012 - 16:30 trong Bất đẳng thức và cực trị

Cho $a$, $b$, $c$ là các số thực dương. Chứng minh rằng:
a) $\sum \frac{b+c}{a}\geq 4\sum ab\sum \frac{1}{a^2}$

a) Mình nghi đề của cậu sai lắm !
Chắc có BĐT này thôi:
$\sum \frac{b+c}{a}\geq \frac{2}{3} \sum ab\sum \frac{1}{a^2}$
Hay đầy đủ hơn là:
${\frac {a+b}{c}}+{\frac {b+c}{a}}+{\frac {c+a}{b}} \geq \frac{2}{3}\, \left( ab+bc+ac \right) \left( {a}^{-2}+{b}^{-2}+{c}^{-2}
\right) $



#345137 Tìm các giá trị của $x$ để $P\left ( x \right )=2002...

Đã gửi bởi nthoangcute on 09-08-2012 - 16:34 trong Đại số

Spam xí: Đề này đúng mà @@ nguyên văn từng dấu câu luôn @@

Tớ nhớ không lầm là $P(x)$ bậc sáu mà ! (Để mất $G(x)$ đi)
Hôm đấy tớ cho thằng ngồi trên tớ chép bài này mà. Tớ chép lại nó câu hình, bị thầy giáo bắt được ! Mệt !



#345153 $a^5b+b^5c+c^5a\leq -3$

Đã gửi bởi nthoangcute on 09-08-2012 - 16:59 trong Bất đẳng thức và cực trị

Với 3 số thực a,b,c thỏa mãn $a+b+c=0$ và $a^2+b^2+c^2=3$.
Chứng minh rằng ta luôn có: $a^5b+b^5c+c^5a\leq -3$

Làm sao mà làm được bài này bạn à !
Dùng Wonfram ta thấy dấu đẳng thức sảy ra khi và chỉ khi:
$\left\{\begin{matrix}
a=-3\,\sqrt {\sqrt [3]{-4+4\,i\sqrt {3}}+4\,{\dfrac {1}{\sqrt [3]{-4+4\,i
\sqrt {3}}}}+4}-4\, \left( \dfrac{1}{4}\,\sqrt [3]{-4+4\,i\sqrt {3}}+{\dfrac {1}
{\sqrt [3]{-4+4\,i\sqrt {3}}}}+1 \right) ^{\dfrac{5}{2}}+10\, \left( \dfrac{1}{4}\,
\sqrt [3]{-4+4\,i\sqrt {3}}+{\dfrac {1}{\sqrt [3]{-4+4\,i\sqrt {3}}}}+1
\right) ^{\dfrac{3}{2}}
\\
b=\dfrac{1}{2}\,\sqrt {\sqrt [3]{-4+4\,i\sqrt {3}}+4\,{\dfrac {1}{\sqrt [3]{-4+4
\,i\sqrt {3}}}}+4}

\\
c=4\, \left( \dfrac{1}{4}\,\sqrt [3]{-4+4\,i\sqrt {3}}+{\dfrac {1}{\sqrt [3]{-4+
4\,i\sqrt {3}}}}+1 \right) ^{\dfrac{5}{2}}-10\, \left( \dfrac{1}{4}\,\sqrt [3]{-4+4\,i
\sqrt {3}}+{\dfrac {1}{\sqrt [3]{-4+4\,i\sqrt {3}}}}+1 \right) ^{\dfrac{3}{2}}+5
/2\,\sqrt {\sqrt [3]{-4+4\,i\sqrt {3}}+4\,{\dfrac {1}{\sqrt [3]{-4+4\,i
\sqrt {3}}}}+4}

\end{matrix}\right.$
_______________________________
Có lẽ nên bỏ lại bài này !



#345296 $\overrightarrow{MD}+\overrightarrow{ME}+...

Đã gửi bởi nthoangcute on 09-08-2012 - 22:14 trong Hình học phẳng

Cho $\triangle ABC$ đều tâm O, M bất kì trong $\triangle$. D,E,F là hình chiếu của M lên BC,CA,AB. Chứng minh: $\overrightarrow{MD}+\overrightarrow{ME}+\overrightarrow{MF}=\frac{3}{2}\overrightarrow{MO}$

Nhìn cái cách kia mà sợ !


Bài làm
Qua M kẻ $I_2I_5;I_1I_4;I_3I_6$ lần lượt song song $BC;AC;AB$
Ta có các tam giác $MI_5I_6;MI_1I_2;MI_3I_4$ là các tam giác đều.
Suy ra : $DI_3=DI_4; FI_1=FI_2;EI_5=EI_6$
Ta có: $\overrightarrow{MI_3}+\overrightarrow{MI_4}=2.\overrightarrow{MD}$
Tương tự: $\overrightarrow{MI_6}+\overrightarrow{MI_5}=2.\overrightarrow{ME}$
$\overrightarrow{MI_1}+\overrightarrow{MI_2}=2.\overrightarrow{MF}$
Cộng vế với vế ta có:
$$\overrightarrow{MD}+\overrightarrow{ME}+\overrightarrow{MF}=\frac{1}{2}.(\overrightarrow{MI_6}+\overrightarrow{MI_5}+\overrightarrow{MI_3}+\overrightarrow{MI_4}+\overrightarrow{MI_1}+\overrightarrow{MI_2})$$
Nên: $\overrightarrow{MD}+\overrightarrow{ME}+\overrightarrow{MF}=\frac{1}{2}(\overrightarrow{MA}+\overrightarrow{MB}+\overrightarrow{MC})=\frac{3}{2}\overrightarrow{MO}$ (tính chất trọng tâm tam giác)




#345303 $$P = \dfrac{2}{a^2 + 1} - \dfrac{2}{b^2 + 1} + \dfr...

Đã gửi bởi nthoangcute on 09-08-2012 - 22:33 trong Bất đẳng thức - Cực trị

Bài toán hay thi $OLYMPIC$ $30-4$
Cho $a, b, c$ thoả mãn :$abc + a + c = b$. Tìm GTLN của :
$$P = \dfrac{2}{a^2 + 1} - \dfrac{2}{b^2 + 1} + \dfrac{3}{c^2 + 1}$$

----------------------------------------
Có thể biến đổi $P$ về dạng hiệu của $\frac{10}{3}$ với một bình phương, khi đó không cần dùng đến đánh giá trên. Và tất nhiên là sẽ cho cùng một kết quả.

Theo lời anh Thành thì:
$$P=\dfrac{10}{3}-\frac{(2c+8ca^2-3a)^2+(2a^2-1)^2}{3(4a^2+1)(a^2+1)(c^2+1)} \leq \frac{10}{3}$$



#345309 $\sum \frac{b^3}{a^2\left ( a^3+2b^3...

Đã gửi bởi nthoangcute on 09-08-2012 - 22:44 trong Bất đẳng thức và cực trị

-Bất đẳng thức phụ của nthoangcute có thể tìm ra bằng Co-si ngược dấu
$\frac{b^3}{a^2(a^3+2b^3)}=\frac{2b^3+a^3-a^3}{2a^2(a^3+2b^3)}=\frac{1}{2a^2}-\frac{a}{2(a^3+2b^3)}\geq \frac{1}{2a^2}-\frac{a}{2.3ab^2}=\frac{1}{2a^2}-\frac{1}{6b^2}$

Spam tị: Thực ra mình dùng cô-si ngược dấu đó !



#345425 Sách giáo khoa điện tử Vật Lý 10

Đã gửi bởi nthoangcute on 10-08-2012 - 11:12 trong Các môn tự nhiên (Vật lý, Hóa học, Sinh học, Công nghệ)

Nguồn: Thuvienvatly.com+Hocmai.vn

Tổng quan: http://thuvienvatly..../sgk-vat-ly-10/
Bài 1: http://thuvienvatly....yen dong co.htm
Bài 2: http://thuvienvatly....g thang deu.htm
Bài 3: http://thuvienvatly....n dong TBDD.htm
Bài 4: http://thuvienvatly....Bai4.Roi TD.htm
Bài 5: http://thuvienvatly..../Bai5.CD TD.htm
Bài 6: http://thuvienvatly....i CD-Cong v.htm
Bài 7: http://thuvienvatly....cua phep do.htm
Bài 8: http://thuvienvatly....m roi tu do.htm
Bài 9: http://thuvienvatly....an tich luc.htm
Bài 10: http://thuvienvatly....i10.Ba DL N.htm
Bài 11: http://thuvienvatly....i11.DL VVHD.htm
Bài 12: http://thuvienvatly....Luc dan hoi.htm
Bài 13: http://thuvienvatly.....Luc ma sat.htm
Bài 14: http://thuvienvatly....c huong tam.htm
Bài 15: http://thuvienvatly....D nem ngang.htm
Bài 16: http://thuvienvatly....e so ma sat.htm
Bài 17: http://thuvienvatly....cua mot vat.htm
Bài 18: http://thuvienvatly....t-Momen luc.htm
Bài 19: http://thuvienvatly.... cung chieu.htm
Bài 20: http://thuvienvatly....Cac dang CB.htm
Bài 21: http://thuvienvatly....ien vat ran.htm
Bài 22: http://thuvienvatly....22.Ngau luc.htm
Bài 23: http://thuvienvatly....ong.DLBT DL.htm
Bài 24: http://thuvienvatly....g-Cong suat.htm
Bài 25: http://thuvienvatly....5.Dong nang.htm
Bài 26: http://thuvienvatly....26.The nang.htm
Bài 27.1: http://thuvienvatly....i27.Co nang.htm
Bài 27.2: http://thuvienvatly....luat Ke-ple.htm
....
Còn nhiều lắm ! Mình học dốt Lý lắm, tìm ở đấy thấy hay hay, lại có nhiều hình ảnh, nhiều flash, bài test của hocmai.vn.
VD:
1. Bài test:
http://thuvienvatly.com/tai-lieu/neohacker/sgk-vat-ly-10/GTDT/De%20KT/Trac%20nghiem%20truc%20tuyen/KIEM%20TRA%20BAI%201/quiz.swf


2. Thí nghiệm:
http://thuvienvatly.com/tai-lieu/neohacker/sgk-vat-ly-10/Lien%20ket%20ngoai%20bai%20hoc/Thi%20nghiem-Video-Flash-Java/Flash/Bai3.dothichuyendongnhanhdandeu(x-t).swf

1

http://thuvienvatly.com/tai-lieu/neohacker/sgk-vat-ly-10/Lien%20ket%20ngoai%20bai%20hoc/Thi%20nghiem-Video-Flash-Java/Flash/Bai3.hinh%203.7.swf

http://thuvienvatly.com/tai-lieu/neohacker/sgk-vat-ly-10/Lien%20ket%20ngoai%20bai%20hoc/Thi%20nghiem-Video-Flash-Java/Flash/Do%20thi%20van%20toc-thoi%20gian.swf

http://thuvienvatly.com/tai-lieu/neohacker/sgk-vat-ly-10/Lien%20ket%20ngoai%20bai%20hoc/Thi%20nghiem-Video-Flash-Java/Flash/CD%20NDD.swf
___________________________
Mọi người thử xem



#345430 CMR: $\frac{-1}{2}\leq \frac{(a+...

Đã gửi bởi nthoangcute on 10-08-2012 - 11:22 trong Bất đẳng thức và cực trị

Bài 1: Chứng minh với mọi số a, b ta đều có:
$\frac{-1}{2}\leq \frac{(a+b).(1-ab)}{(1+a^{2}).(1+b^{2})}\leq \frac{1}{2}$.

Bài 2: Cho $x^{2}+y^{2}> 0$, chứng minh:
$-2\sqrt{2}-2\leq \frac{x^{2}-(x-4y)^{2}}{x^{2}+4y^{2}}\leq 2\sqrt{2}-2.$

Bài 1:
$\frac{(a+b).(1-ab)}{(1+a^{2}).(1+b^{2})}+\frac{1}{2}=\frac{(ab-a-b-1)^2}{2(1+a^2)(1+b^2)} \geq 0$
$\frac{1}{2}-\frac{(a+b).(1-ab)}{(1+a^{2}).(1+b^{2})} =\frac{(ab+a+b-1)^2}{2(1+a^2)(1+b^2)} \geq 0$
Bài 2:
$\frac{x^{2}-(x-4y)^{2}}{x^{2}+4y^{2}}+2\sqrt{2}+2=\frac{2(\sqrt{2}+1)(x-2y+2\sqrt{2}y)^2}{x^2+4y^2} \geq 0$
$2\sqrt{2}-2-\frac{x^{2}-(x-4y)^{2}}{x^{2}+4y^{2}}=\frac{2(\sqrt{2}-1)(2y-x+2\sqrt{2}y)^2}{x^2+4y^2} \geq 0$
Suy ra đpcm
_______________
P/s: Cách mình ngắn nhất !



#345523 $\left\{\begin{matrix} x^{4}+4x^...

Đã gửi bởi nthoangcute on 10-08-2012 - 15:44 trong Phương trình - hệ phương trình - bất phương trình

Đặt $t=y^{2}$ ta có hệ

$\left\{\begin{matrix}t-4y= 2-x^{4}-4x^{2}\\ y(x^{2}+6)+6y=23 –x^{2}\end{matrix}\right.$
Ta có $D=x^{2}+6,Dt=-x^{6}-10x^{4}-30x^{2}+104,Dy=23-2x^{2}$
Lại có $\dfrac{Dt}{D}=(\dfrac{Dy}{D})^{2}$
$\Rightarrow (x^{2}+6)(2-x^{4}-4x^{2})=(23-2x^{2})^{2}$
.................
$ \Leftrightarrow ( 1-x)(x+1)(x^{2}+1)(x^{4}+16x^{2}+95)=0$
$ \Leftrightarrow x= \pm 1$
$ \Rightarrow y=3$
Vậy hệ pt có 2 nghiệm $(x;y)=(1;3);(-1;3)$
$P/s$ : có lẻ đâu nhỉ?

Dù rất cố gắng nhưng bạn đã làm sai !
________________________________
Chi tiết nhé:
1. $(x;y)=(1;3);(-1;3)$ không thỏa mãn đề bài.
VD: $(x;y)=(1;3)$ thì $yx^2+x^2+6y = 22$ nên không thỏa mãn
2. Nghiệm là: $\{x = 0,965717935477349, y = 3,18312803839574\}$
3. Nghiệm chính xác là:
$x$ là nghiệm dương của phương trình : $k^8+16k^6+87k^4+6k^2-95=0$
$y=-\dfrac{27}{29}x^2+\dfrac{127}{29}-\dfrac{x^6}{29}-\dfrac{10}{29}x^4$
4. Chính xác hơn nữa là:
$x=\sqrt{-2+\dfrac{1}{6}\,\sqrt {3}\sqrt{\dfrac{-126\sqrt[3]{855657+834\sqrt{695679}}+\sqrt[3]{855657+834\sqrt{695679}}^2+6285}{\sqrt[3]{855657+834\sqrt{695679}}}}+\dfrac{1}{6}\,\sqrt {{\dfrac {-756\,\sqrt[3]{855657+834\sqrt{695679}}\sqrt{\dfrac{-126\sqrt[3]{855657+834\sqrt{695679}}+\sqrt[3]{855657+834\sqrt{695679}}^2+6285}{\sqrt[3]{855657+834\sqrt{695679}}}}-3\,\sqrt{\dfrac{-126\sqrt[3]{855657+834\sqrt{695679}}+\sqrt[3]{855657+834\sqrt{695679}}^2+6285}{\sqrt[3]{855657+834\sqrt{695679}}}}{\sqrt[3]{855657+834\sqrt{695679}}}^{2}-18855\,\sqrt{\dfrac{-126\sqrt[3]{855657+834\sqrt{695679}}+\sqrt[3]{855657+834\sqrt{695679}}^2+6285}{\sqrt[3]{855657+834\sqrt{695679}}}}
+5004\,\sqrt {3}\sqrt[3]{855657+834\sqrt{695679}}}{\sqrt[3]{855657+834\sqrt{695679}}\sqrt{\dfrac{-126\sqrt[3]{855657+834\sqrt{695679}}+\sqrt[3]{855657+834\sqrt{695679}}^2+6285}{\sqrt[3]{855657+834\sqrt{695679}}}}}}}}$
Và $y=...$

Cậu xem lại cách làm của mình đi ! Thực ra nếu đề là $22$ thì rất dễ !


Hình đã gửi

http://farm9.staticf...232eeb3f4_k.jpg



#345628 Trang chủ bị lỗi ?

Đã gửi bởi nthoangcute on 10-08-2012 - 21:17 trong Góp ý cho diễn đàn

Máy mình cũng bị rồi ! (Bị suốt, từ 2 ngày trước)
Mình thì tự biết cách sửa rồi (Dùng adblock, ấn chuột phải vào trang đó, chọn Block this ad là tự khắc nó chuyển lại bình thường)
Không biết là do gì nữa đây...



#345667 Bức ảnh ngộ nghĩnh về bản đồ thế giới

Đã gửi bởi nthoangcute on 10-08-2012 - 22:56 trong Quán hài hước

Nó nặng 2,7 MB, phóng to ra đẹp cực !

Image
Format : JPEG
Width : 4 736 pixels
Height : 2 188 pixels
Chroma subsampling : 4:4:4
Bit depth : 8 bits
Compression mode : Lossy

Hình đã gửi



#345678 Bức ảnh ngộ nghĩnh về bản đồ thế giới

Đã gửi bởi nthoangcute on 10-08-2012 - 23:21 trong Quán hài hước

Vn: We kicked Mongol, Japan, France, US and your @$$ Hình đã gửi

Cái đấy không hiểu nghĩa là gì nữa !



#345750 Chứng minh " $A = \frac{12}{1.4.7}+\fr...

Đã gửi bởi nthoangcute on 11-08-2012 - 10:18 trong Bất đẳng thức và cực trị

( Lớp 6) . Chứng minh rằng $A = \frac{12}{1.4.7}+\frac{12}{4.7.10}+...\frac{12}{54.57.60}< \frac{1}{2}$

Số hạng tổng quát của cái này là gì vậy ? (Số đầu và số cuối không như nhau) !
Có phải $\frac{12}{(3n+1)(3n+4)(3n+7)}$
Nhưng mà $54$ chia hết cho $3$ mất rồi !



#345757 Tìm giá trị lớn nhất của biểu thức: A = $\begin{vmatrix}...

Đã gửi bởi nthoangcute on 11-08-2012 - 10:22 trong Bất đẳng thức và cực trị

Cho 3 số thực x,y,z thỏa mãn $x^{2} + y^{2} + z^{2} = 1$. Tìm giá trị lớn nhất của biểu thức: A = $\begin{vmatrix} x + 2y + 3z\\ \end{vmatrix}$

Áp dụng Bunyakovsky ta được:
$A^2=(x + 2y + 3z)^2 \leq (1+4+9)(x^2+y^2+z^2)=14$
Suy ra $A_{max}=\sqrt{14}$



#345759 Chứng minh " $A = \frac{12}{1.4.7}+\fr...

Đã gửi bởi nthoangcute on 11-08-2012 - 10:24 trong Bất đẳng thức và cực trị

Ta có công thức :
$\frac{3}{(n-3)n(n+3)} =\frac{1}{2}.(\frac{1}{(n-3)n} -\frac{1}{n(n+3)}$
Thay vào ta có :
$A =2(\frac{1}{1.4}-\frac{1}{4.7}+\frac{1}{4.7} -\frac{1}{7.10} +.....+\frac{1}{54.57} -\frac{1}{57.60})$
$A =2(\frac{1}{4} -\frac{1}{57.60})$
$Q.E.D \leftrightarrow 2(\frac{1}{4} -\frac{1}{57.60}) < \frac{1}{2}$
$\leftrightarrow \frac{1}{4} -\frac{1}{57.60} < \frac{1}{4} :\text{hiển nhiên} \rightarrow DPCM$

Riêng số hạng tổng quát đã sai rồi kìa !
Thôi đợi chủ thớt kiểm định chất lượng đề bài đã



#345761 \[\begin{cases} & \text{ } x + y^...

Đã gửi bởi nthoangcute on 11-08-2012 - 10:28 trong Phương trình - hệ phương trình - bất phương trình

Bài 1
\[\begin{cases}
& \text{ } x + y^{3}=2xy^{2} \\
& \text{ } x^{3}+y^{9}= 2xy^{4}
\end{cases}\]

Từ giả thiết ta có:
$(x+y^3)^3-(2xy^2)^3-x^3-y^9+2xy^4-3xy^3(x+y^3-2xy^2)=0$
Suy ra $2xy^4(4xy+1)(xy-1)=0$
Đến đây dễ rồi nhỉ !



#345763 \[\begin{cases} & \text{ } x + y^...

Đã gửi bởi nthoangcute on 11-08-2012 - 10:34 trong Phương trình - hệ phương trình - bất phương trình

Bài 2: Giải phương trình: Khánh Hòa ( 08/09)
\[\frac{x^{2}}{x-1}+\sqrt{x-1} +\frac{\sqrt{x-1}}{x^{2}}=\frac{x-1}{x^{2}}+\frac{1}{\sqrt{x-1}}+\frac{x^{2}}{\sqrt{x-1}}\]

Đặt $x=y^2+1$
Phương trình đã cho tương đương với:
${\frac { \left( {y}^{2}+1 \right) ^{2}}{{y}^{2}}}+y+{\frac {y}{
\left( {y}^{2}+1 \right) ^{2}}}={\frac {{y}^{2}}{ \left( {y}^{2}+1
\right) ^{2}}}+\frac{1}{y}+{\frac { \left( {y}^{2}+1 \right) ^{2}}{y}}$
$\Leftrightarrow {\frac { \left( y-1 \right) \left( {y}^{2}+y+1 \right) \left( {y}^{2
}-y+1 \right) \left( {y}^{4}+2\,{y}^{2}-y+1 \right) }{{y}^{2} \left(
{y}^{2}+1 \right) ^{2}}}=0$
$\Leftrightarrow y=1$
$\Leftrightarrow x=2$



#345769 Tìm giá trị nhỏ nhất của biểu thức : A = $x^{4} + y^{4...

Đã gửi bởi nthoangcute on 11-08-2012 - 10:43 trong Bất đẳng thức và cực trị

Cho 3 số x,y,z thay đổi luôn thỏa mãn điều kiện xy + yz + xz = 4. Tìm giá trị nhỏ nhất của biểu thức : A = $x^{4} + y^{4} + z^{4}$

Áp dụng Holder ta được:
$A^2(1+1+1)^2 \geq (xy+yz+zx)^4=4^4$
Suy ra $A \geq \frac{16}{3}$